An integral resulting in harmonic number

Calculus (Integrals, Series)
Post Reply
User avatar
Tolaso J Kos
Administrator
Administrator
Posts: 867
Joined: Sat Nov 07, 2015 6:12 pm
Location: Larisa
Contact:

An integral resulting in harmonic number

#1

Post by Tolaso J Kos »

Let $m \in \mathbb{N}$. Prove that:

$$\int_0^1 x^m \log(1-x)\, {\rm d}x=-\frac{\mathcal{H}_{m+1}}{m+1}$$
Generalization
Can you find an explicit formula for the integral

$$\int_0^1 x^m \log^n (1-x)\, {\rm d}x$$

if $m,n \in \mathbb{N}$. Stirling numbers will come in handy.
Imagination is much more important than knowledge.
User avatar
Riemann
Posts: 178
Joined: Sat Nov 14, 2015 6:32 am
Location: Melbourne, Australia

Re: An integral resulting in harmonic number

#2

Post by Riemann »

Tolaso J Kos wrote:Let $m \in \mathbb{N}$. Prove that:

$$\int_0^1 x^m \log(1-x)\, {\rm d}x=-\frac{\mathcal{H}_{m+1}}{m+1}$$
Here is a solution. Recalling the fact that:

\begin{equation} \mathcal{H}_m = \sum_{k=1}^{\infty} \left [ \frac{1}{k} - \frac{1}{m+k} \right ] \end{equation}
Proposition: It holds that:

$$\mathcal{H}_m = \sum_{k=1}^{\infty} \left [ \frac{1}{k} - \frac{1}{m+k} \right ]$$

Proof:

\begin{align*}
\mathcal{H}_m &= \int_{0}^{1}\frac{1-x^m}{1-x}\, {\rm d}t\\
&= \int_{0}^{1}\left ( 1-x^m \right ) \sum_{k=0}^{\infty} x^k \, {\rm d}x\\
&= \sum_{k=0}^{\infty} \int_{0}^{1}\left ( x^k- x^{m+k} \right )\, {\rm d}x\\
&= \sum_{k=0}^{\infty} \left [ \frac{1}{k+1} - \frac{1}{m+k+1} \right ] \\
&= \sum_{k=1}^{\infty} \left [ \frac{1}{k} - \frac{1}{m+k} \right ]
\end{align*}
Now for our integral we have successively:

\begin{align*}
\mathcal{J} &=\int_{0}^{1} x^m \log(1-x) \, {\rm d}x \\
&= - \int_{0}^{1}x^m \sum_{n=1}^{\infty} \frac{x^n}{n} \, {\rm d}x \\
&= - \sum_{n=1}^{\infty} \frac{1}{n} \int_{0}^{1}x^{m+n} \, {\rm d}x\\
&= - \sum_{n=1}^{\infty} \frac{1}{n \left ( n+m+1 \right )}\\
&= - \sum_{n=1}^{\infty}\left[\frac{1}{(m+1)n} - \frac{1}{(m+1) \left ( m+n+1 \right )} \right]\\
&= - \frac{1}{m+1} \sum_{n=1}^{\infty} \left [ \frac{1}{n} - \frac{1}{n+m+1} \right ]\\
&\overset{(1)}{=} - \frac{\mathcal{H}_{m+1}}{m+1}
\end{align*}

For the generalization I seem to recall that $\log^n (1-x)$ has a Taylor series of the form involving stirling numbers and that the sum begins from $n$, that is $k=n$ to $\infty$ but I cannot seem to remember the exact formula .. Anyone wishes to try or at least present the formula ?
$\displaystyle \sum_{n=1}^{\infty}\frac{1}{n^s}= \prod_{p \; \text{prime}}\frac{1}{1-p^{-s}}$
Post Reply

Create an account or sign in to join the discussion

You need to be a member in order to post a reply

Create an account

Not a member? register to join our community
Members can start their own topics & subscribe to topics
It’s free and only takes a minute

Register

Sign in

Who is online

Users browsing this forum: No registered users and 40 guests